An algebra problem by Christian Daang

Algebra Level 5

n = 4 1 ( n 2 ) ( n 1 ) n ( n + 1 ) ( n + 2 ) \large \sum_{n = 4}^\infty \dfrac{1}{(n-2)(n-1)n(n+1)(n+2)}

The infinite sum above can be expressed as a b \dfrac{a}{b} , where a a and b b are coprime positive integers, find a + b a + b .


The answer is 481.

This section requires Javascript.
You are seeing this because something didn't load right. We suggest you, (a) try refreshing the page, (b) enabling javascript if it is disabled on your browser and, finally, (c) loading the non-javascript version of this page . We're sorry about the hassle.

4 solutions

Yatin Khanna
Feb 17, 2017

n = 4 1 ( n 2 ) ( n 1 ) n ( n + 1 ) ( n + 2 ) \displaystyle \sum_{n=4}^\infty \frac{1}{(n-2)(n-1)n (n+1)(n+2)}
= 1 4 n = 4 ( n + 2 ) ( n 2 ) ( n 2 ) ( n 1 ) n ( n + 1 ) ( n + 2 ) = \frac{1}{4} \displaystyle \sum_{n=4}^\infty \frac{(n+2)-(n-2)}{(n-2)(n-1) n (n+1) (n+2)}
= 1 4 n = 4 ( 1 ( n 2 ) ( n 1 ) n ( n + 1 ) 1 ( n 1 ) n ( n + 1 ) ( n + 2 ) ) = \frac {1}{4} \displaystyle \sum_{n=4}^\infty\left( \frac {1}{(n-2)(n-1) n (n+1)} - \frac {1}{(n-1)n (n+1)(n+2)}\right)

On observing we can see that on expanding the sum; everything cancels out except 1 2 3 4 5 \frac {1}{2*3*4*5}
So, the sum is 1 4 1 2 3 4 5 = 1 480 \frac {1}{4} * \frac {1}{2*3*4*5} = \frac {1}{480}
And our answer is 1 + 480 = 481 1+480=\boxed {481}

Very simple

I Gede Arya Raditya Parameswara - 4 years, 3 months ago

I did the same way.

Niranjan Khanderia - 4 years, 1 month ago
Chew-Seong Cheong
Feb 17, 2017

Similar to Christian Daang 's approach.

S = n = 4 1 ( n 2 ) ( n 1 ) n ( n + 1 ) ( n + 2 ) By partial fractions – see below. = n = 4 ( 1 24 ( n 2 ) 1 6 ( n 1 ) + 1 4 n 1 6 ( n + 1 ) + 1 24 ( n 2 ) ) = 1 24 n = 4 ( 1 n 2 4 n 1 + 6 n 4 n + 1 + 1 n 2 ) = 1 24 n = 4 ( 1 n 2 1 n 1 3 n 1 + 3 n + 3 n 3 n + 1 1 n + 1 + 1 n 2 ) = 1 24 ( 1 2 3 3 + 3 4 1 5 ) = 1 24 ( 10 20 + 15 4 20 ) = 1 480 \begin{aligned} S & = \sum_{n=4}^\infty \frac 1{(n-2)(n-1)n(n+1)(n+2)} \quad \quad \quad \quad \small \color{#3D99F6} \text{By partial fractions -- see below.} \\ & = \sum_{n=4} ^\infty \left( \frac 1{24(n-2)} - \frac 1{6(n-1)} + \frac 1{4n} - \frac 1{6(n+1)} + \frac 1{24(n-2)} \right) \\ & = \frac 1{24} \sum_{n=4} ^\infty \left( \frac 1{n-2} - \frac 4{n-1} + \frac 6{n} - \frac 4{n+1} + \frac 1{n-2} \right) \\ & = \frac 1{24} \sum_{n=4} ^\infty \left({\color{#3D99F6}\frac 1{n-2} - \frac 1{n-1}}{\color{#D61F06} - \frac 3{n-1} + \frac 3{n}}{\color{#3D99F6}+ \frac 3{n} - \frac 3{n+1}}{\color{#D61F06} - \frac 1{n+1} + \frac 1{n-2}} \right) \\ & = \frac 1{24} \left({\color{#3D99F6}\frac 12}{\color{#D61F06} - \frac 33}{\color{#3D99F6}+ \frac 34}{\color{#D61F06} - \frac 15} \right) \\ & = \frac 1{24} \left(\frac {10-20+15-4}{20} \right) \\ & = \frac 1{480} \end{aligned}

a + b = 1 + 480 = 481 \implies a + b = 1 + 480 = \boxed{481}


Partial fractions:

Let 1 ( n 2 ) ( n 1 ) n ( n + 1 ) ( n + 2 ) = A n 2 + B n 1 + C n + D n + 1 + E n + 2 \dfrac 1{(n-2)(n-1)n(n+1)(n+2)} = \dfrac A{n-2} + \dfrac B{n-1} + \dfrac Cn + \dfrac D{n+1} + \dfrac E{n+2} .

Multiplying both sides with ( n 2 ) ( n 1 ) n ( n + 1 ) ( n + 2 ) (n-2)(n-1)n(n+1)(n+2) , we get:

1 = A ( n 1 ) n ( n + 1 ) ( n + 2 ) + B ( n 2 ) n ( n + 1 ) ( n + 2 ) + C ( n 2 ) ( n 1 ) ( n + 1 ) ( n + 2 ) + D ( n 2 ) ( n 1 ) n ( n + 2 ) + E ( n 2 ) ( n 1 ) n ( n + 1 ) \small 1 = A(n-1)n(n+1)(n+2) + B{\color{#D61F06}(n-2)}n(n+1)(n+2) + C{\color{#D61F06}(n-2)}(n-1)(n+1)(n+2) + D{\color{#D61F06}(n-2)}(n-1)n(n+2) + E{\color{#D61F06}(n-2)}(n-1)n(n+1)

If we put n = 2 {\color{#D61F06}n=2} , then we have 1 = A ( 2 1 ) 2 ( 2 + 1 ) ( 2 + 2 ) + B ( 0 ) + C ( 0 ) + D ( 0 ) + E ( 0 ) 1=A(2-1)2(2+1)(2+2) + B{\color{#D61F06}(0)}+C{\color{#D61F06}(0)}+D{\color{#D61F06}(0)}+E{\color{#D61F06}(0)} .

A n 2 = 1 ( n 2 ) ( n 1 ) n ( n + 1 ) ( n + 2 ) n = 2 = 1 ( 2 1 ) ( 2 ) ( 2 + 1 ) ( 2 + 2 ) = 1 ( 1 ) ( 2 ) ( 3 ) ( 4 ) = 1 24 B n 1 = 1 ( n 2 ) ( n 1 ) n ( n + 1 ) ( n + 2 ) n = 1 = 1 ( 1 2 ) ( 1 ) ( 1 + 1 ) ( 1 + 2 ) = 1 ( 1 ) ( 1 ) ( 2 ) ( 3 ) = 1 6 C n = 1 ( n 2 ) ( n 1 ) n ( n + 1 ) ( n + 2 ) n = 0 = 1 ( 0 2 ) ( 0 1 ) ( 0 + 1 ) ( 0 + 2 ) = 1 ( 2 ) ( 1 ) ( 1 ) ( 2 ) = 1 4 D n + 1 = 1 ( n 2 ) ( n 1 ) n ( n + 1 ) ( n + 2 ) n = 1 = 1 ( 1 2 ) ( 1 1 ) ( 1 ) ( 1 + 2 ) = 1 ( 3 ) ( 2 ) ( 1 ) ( 1 ) = 1 6 E n + 2 = 1 ( n 2 ) ( n 1 ) n ( n + 1 ) ( n + 2 ) n = 2 = 1 ( 2 2 ) ( 2 1 ) ( 2 ) ( 2 + 1 ) = 1 ( 4 ) ( 3 ) ( 2 ) ( 1 ) = 1 24 \small \begin{aligned} \implies \frac A{\color{#D61F06} \cancel{n-2}} & = \frac 1{({\color{#D61F06} \cancel{n-2}})(n-1)n(n+1)(n+2)} \bigg|_{\color{#D61F06}n=2} = \frac 1{(2-1)(2)(2+1)(2+2)} = \frac 1{(1)(2)(3)(4)} = \frac 1{24} \\ \frac B{\color{#D61F06} \cancel{n-1}} & = \frac 1{(n-2)({\color{#D61F06} \cancel{n-1}})n(n+1)(n+2)} \bigg|_{\color{#D61F06}n=1} = \frac 1{(1-2)(1)(1+1)(1+2)} = \frac 1{(-1)(1)(2)(3)} = - \frac 1 6 \\ \frac C{\color{#D61F06} \cancel n} & = \frac 1{(n-2)(n-1){\color{#D61F06} \cancel{n}}(n+1)(n+2)} \bigg|_{\color{#D61F06}n=0} = \frac 1{(0-2)(0-1)(0+1)(0+2)} = \frac 1{(-2)(-1)(1)(2)} = \frac 14 \\ \frac D{\color{#D61F06} \cancel{n+1}} & = \frac 1{(n-2)(n-1)n{(\color{#D61F06} \cancel{n+1}}) (n+2)} \bigg|_{\color{#D61F06}n=-1} = \frac 1{(-1-2)(-1-1)(-1)(-1+2)} = \frac 1{(-3)(-2)(-1)(1)} = - \frac 16 \\ \frac E{\color{#D61F06} \cancel {n+2}} & = \frac 1{(n-2)(n-1)n(n+1){(\color{#D61F06} \cancel{n+2}})} \bigg|_{\color{#D61F06}n=-2} = \frac 1{(-2-2)(-2-1)(-2)(-2+1)} = \frac 1{(-4)(-3)(-2)(-1)} = \frac 1{24} \end{aligned}

Sir, can you help me to prove that 1 ( n 2 ) ( n 1 ) ( n ) ( n + 1 ) ( n + 2 ) = 1 24 ( n 2 ) 1 6 ( n 1 ) + 1 4 n 1 6 ( n 1 ) + 1 24 ( n 2 ) \dfrac{1}{(n-2)(n-1)(n)(n+1)(n+2)} = \dfrac{1}{24(n-2)} - \dfrac{1}{6(n-1)} + \dfrac{1}{4n} - \dfrac{1}{6(n-1)} + \dfrac{1}{24(n-2)} ?

Fidel Simanjuntak - 4 years, 3 months ago

Log in to reply

I have added the explanation as note.

Chew-Seong Cheong - 4 years, 3 months ago

Log in to reply

Thank you sir. Very nice solution!

Fidel Simanjuntak - 4 years, 3 months ago

Log in to reply

@Fidel Simanjuntak This is the link.
link text

Niranjan Khanderia - 4 years, 1 month ago

Does it work for another expression? For example 1 ( n + 3 ) ( n + 4 ) ( n + 5 ) ( n + 6 ) \dfrac{1}{(n+3)(n+4)(n+5)(n+6)}

Fidel Simanjuntak - 4 years, 3 months ago

Log in to reply

@Fidel Simanjuntak Yes. Why don't you work it out to find out. You learn better that way.

Chew-Seong Cheong - 4 years, 3 months ago
Christian Daang
Feb 16, 2017

Applying Partial Fractions, we can simplify it to:

n = 4 ( 1 ( n 2 ) ( n 1 ) ( n ) ( n + 1 ) ( n + 2 ) ) = n = 4 ( 1 ( n 2 ) ( 24 ) 1 ( n 1 ) ( 6 ) + 1 ( n ) ( 4 ) 1 ( n + 1 ) ( 6 ) + 1 ( n + 2 ) ( 24 ) ) = 1 24 n = 4 ( 1 ( n 2 ) 4 ( n 1 ) + 6 ( n ) 4 ( n + 1 ) + 1 ( n + 2 ) ) = 1 24 ( 1 2 4 3 + 6 4 4 5 + 1 6 1 3 4 4 + 6 5 4 6 + 1 7 1 4 4 5 + 6 6 4 7 + 1 8 1 5 4 6 + 6 7 4 8 + 1 9 1 6 4 7 + 6 8 4 9 + 1 10 1 7 4 8 + 6 9 4 10 + 1 11 1 ( n 2 ) 4 ( n 1 ) + 6 ( n ) 4 ( n + 1 ) + 1 ( n + 2 ) ) \begin{aligned} \sum_{n = 4}^\infty \left( \cfrac{1}{(n-2)(n-1)(n)(n+1)(n+2)} \right) & = \sum_{n = 4}^\infty \left( \cfrac{1}{(n-2)(24)} - \cfrac{1}{(n-1)(6)} + \cfrac{1}{(n)(4)} - \cfrac{1}{(n+1)(6)} + \cfrac{1}{(n+2)(24)} \right) \\ & = \cfrac{1}{24} \cdot \sum_{n = 4}^\infty \left( \cfrac{1}{(n-2)} - \cfrac{4}{(n-1)} + \cfrac{6}{(n)} - \cfrac{4}{(n+1)} + \cfrac{1}{(n+2)} \right ) \\ & = \cfrac{1}{24} \cdot \begin{pmatrix} \cfrac{1}{2} & - \cfrac{4}{3} & + \cfrac{6}{4} & - \cfrac{4}{5} & \color{#3D99F6} \cancel {+ \cfrac{1}{6}} \\ \cfrac{1}{3} & - \cfrac{4}{4} & + \cfrac{6}{5} & \color{#3D99F6} \cancel {- \cfrac{4}{6}} & \color{#3D99F6} \cancel {+ \cfrac{1}{7}} \\ \cfrac{1}{4} & - \cfrac{4}{5} & \color{#3D99F6} \cancel {+\cfrac{6}{6}} & \color{#3D99F6} \cancel {- \cfrac{4}{7}} & \color{#3D99F6} \cancel {+ \cfrac{1}{8}} \\ \cfrac{1}{5} & \color{#3D99F6} \cancel {- \cfrac{4}{6}} & \color{#3D99F6} \cancel {+ \cfrac{6}{7}} & \color{#3D99F6} \cancel {- \cfrac{4}{8}} & \color{#3D99F6} \cancel {+ \cfrac{1}{9}} \\ \color{#3D99F6} \cancel {\cfrac{1}{6}} & \color{#3D99F6} \cancel {- \cfrac{4}{7}} & \color{#3D99F6} \cancel {+ \cfrac{6}{8}} & \color{#3D99F6} \cancel {- \cfrac{4}{9}} & \color{#3D99F6} \cancel {+ \cfrac{1}{10}} \\ \color{#3D99F6} \cancel {\cfrac{1}{7}} & \color{#3D99F6} \cancel {- \cfrac{4}{8}} & \color{#3D99F6} \cancel {+ \cfrac{6}{9}} & \color{#3D99F6} \cancel {- \cfrac{4}{10}} & \color{#3D99F6} \cancel {+ \cfrac{1}{11}} \\ \vdots & \vdots & \vdots & \vdots & \vdots \\ \color{#3D99F6} \cancel {\cfrac{1}{(n-2)}} & - \cfrac{4}{(n-1)} & + \cfrac{6}{(n)} & - \cfrac{4}{(n+1)} & + \cfrac{1}{(n+2)} \end{pmatrix} \end{aligned}

As you notice, as the pattern will continues, the numbers colored in blue will be cancelled out and the sum will be: 1 24 ( 1 2 4 3 + 1 3 + 6 4 4 4 + 1 4 4 5 + 6 5 4 5 + 1 5 ) 4 ( n 1 ) + 6 ( n ) 4 ( n + 1 ) + 1 ( n + 2 ) \cfrac{1}{24} \cdot \left( \cfrac{1}{2} - \cfrac{4}{3} + \cfrac{1}{3} + \cfrac{6}{4} - \cfrac{4}{4} + \cfrac{1}{4} - \cfrac{4}{5} + \cfrac{6}{5} - \cfrac{4}{5} + \cfrac{1}{5} \right) - \cfrac{4}{(n-1)} + \cfrac{6}{(n)} - \cfrac{4}{(n+1)} + \cfrac{1}{(n+2)}

which will converge to:

1 24 ( 1 2 4 3 + 1 3 + 6 4 4 4 + 1 4 4 5 + 6 5 4 5 + 1 5 ) = 1 480 a + b = 481 \cfrac{1}{24} \cdot \left( \cfrac{1}{2} - \cfrac{4}{3} + \cfrac{1}{3} + \cfrac{6}{4} - \cfrac{4}{4} + \cfrac{1}{4} - \cfrac{4}{5} + \cfrac{6}{5} - \cfrac{4}{5} + \cfrac{1}{5} \right) = \cfrac{1}{480} \\ \implies \boxed{a + b = 481}

Aditya Dhawan
Feb 26, 2017

n = 4 1 ( n 2 ) ( n 1 ) n ( n + 1 ) ( n + 2 ) = 1 4 1 ( n 2 ) ( n 1 ) ( n ) ( n + 1 ) 1 ( n 1 ) n ( n + 1 ) ( n + 2 ) = 1 4.2.3.4.5 = 1 480 { k = a n f ( k ) f ( k + 1 ) = f ( a ) f ( n + 1 ) } \sum _{ n=4 }^{ \infty }{ \frac { 1 }{ (n-2)(n-1)n(n+1)(n+2) } } =\quad \frac { 1 }{ 4 } \sum { \frac { 1 }{ (n-2)(n-1)(n)(n+1) } } -\frac { 1 }{ (n-1)n(n+1)(n+2) } =\frac { 1 }{ 4.2.3.4.5 } =\boxed { \frac { 1 }{ 480 } } \\ \{ \because \sum _{ k=a }^{ n }{ f\left( k \right) } -f\left( k+1 \right) \quad =\quad f\left( a \right) -f\left( n+1 \right) \}

0 pending reports

×

Problem Loading...

Note Loading...

Set Loading...